interesting function equation (fe) in IR
by skellyrah, Apr 23, 2025, 9:51 AM
find all function F: IR->IR such that 

2^x+3^x = yx^2
by truongphatt2668, Apr 22, 2025, 3:38 PM
Prove that the following equation has infinite integer solutions:


FE solution too simple?
by Yiyj1, Apr 9, 2025, 3:26 AM
Find all functions
such that the equality
holds for all pairs of real numbers
.
My solution
I feel like my solution is too simple. Is there something I did wrong or something I missed?



My solution
Clearly,
is an obvious solution. Now, let
. Then, we have
or
. Therefore, the solutions are
.





I feel like my solution is too simple. Is there something I did wrong or something I missed?
Divisibility on 101 integers
by BR1F1SZ, Aug 9, 2024, 12:31 AM
There are
positive integers
such that for every index
, with
,
is a multiple of
. Determine the greatest possible value of the largest of the
numbers.







This post has been edited 2 times. Last edited by BR1F1SZ, Jan 27, 2025, 5:01 PM
Z[x], P(\sqrt[3]5+\sqrt[3]25)=5+\sqrt[3]5
by jasperE3, May 31, 2021, 4:28 PM
Prove that there is no polynomial
with integer coefficients such that
.

![$P\left(\sqrt[3]5+\sqrt[3]{25}\right)=5+\sqrt[3]5$](http://latex.artofproblemsolving.com/5/8/e/58ec2a921cd988bd5ae017a8653544ddd758225f.png)
IMO Shortlist 2011, G4
by WakeUp, Jul 13, 2012, 11:41 AM
Let
be an acute triangle with circumcircle
. Let
be the midpoint of
and let
be the midpoint of
. Let
be the foot of the altitude from
and let
be the centroid of the triangle
. Let
be a circle through
and
that is tangent to the circle
at a point
. Prove that the points
and
are collinear.
Proposed by Ismail Isaev and Mikhail Isaev, Russia

















Proposed by Ismail Isaev and Mikhail Isaev, Russia
Find all sequences satisfying two conditions
by orl, Jul 13, 2008, 1:21 PM
Let
be an integer. Find all sequences
satisfying the following conditions:
![\[ \text{ (a) } a_i \in \left\{0,1\right\} \text{ for all } 1 \leq i \leq n^2 + n;
\]](//latex.artofproblemsolving.com/3/c/5/3c509ec2e9013e8d3be492c8eb44a7c33841b74e.png)
![\[ \text{ (b) } a_{i + 1} + a_{i + 2} + \ldots + a_{i + n} < a_{i + n + 1} + a_{i + n + 2} + \ldots + a_{i + 2n} \text{ for all } 0 \leq i \leq n^2 - n.
\]](//latex.artofproblemsolving.com/9/7/d/97d2a467d1c0dc8594ec024c3bb9b8c87ee85b19.png)
Author: Dusan Dukic, Serbia


![\[ \text{ (a) } a_i \in \left\{0,1\right\} \text{ for all } 1 \leq i \leq n^2 + n;
\]](http://latex.artofproblemsolving.com/3/c/5/3c509ec2e9013e8d3be492c8eb44a7c33841b74e.png)
![\[ \text{ (b) } a_{i + 1} + a_{i + 2} + \ldots + a_{i + n} < a_{i + n + 1} + a_{i + n + 2} + \ldots + a_{i + 2n} \text{ for all } 0 \leq i \leq n^2 - n.
\]](http://latex.artofproblemsolving.com/9/7/d/97d2a467d1c0dc8594ec024c3bb9b8c87ee85b19.png)
Author: Dusan Dukic, Serbia
This post has been edited 2 times. Last edited by orl, Jan 4, 2009, 8:47 PM
Prove perpendicular
by shobber, Apr 1, 2006, 10:42 AM
Let
be a triangle. Let
and
be the points in which the median and the angle bisector, respectively, at
meet the side
. Let
and
be the points in which the perpendicular at
to
meets
and
, respectively. And
the point in which the perpendicular at
to
meets
produced.
Prove that
is perpendicular to
.















Prove that


IMO problem 1
by iandrei, Jul 14, 2003, 5:06 PM
Let
be a
-element subset of the set
. Prove that there exist numbers
,
in
such that the sets
are pairwise disjoint.






![\[ A_j=\{x+t_j\mid x\in A\},\qquad j=1,2,\ldots,100 \]](http://latex.artofproblemsolving.com/d/5/c/d5c2a7b943e8c7200361698f3eb6f83700ad68d6.png)
"Where wisdom and valor fail, all that remains is faith. . . And it can overcome all." -Toa Mata Tahu
Archives





































































Shouts
Submit
536 shouts
Tags
About Owner
- Posts: 3075
- Joined: Dec 24, 2011
Blog Stats
- Blog created: Jan 14, 2012
- Total entries: 600
- Total visits: 1582000
- Total comments: 771
Search Blog